Convolution Problem: Solve 2e^{-t}u(t) \ast e^{t}u(-t)

In summary, the homework statement is trying to find an equation for the integral of a step function over an interval, but is having trouble understanding the second function. The attempted solution suggests solving for the limits of the integral using the Fourier transform, and remembering that the second function only exists for values of \tau greater than t. Finally, the student asks how to do the integration manually.
  • #1
jegues
1,097
3

Homework Statement



Convolve,

[tex]2e^{-t}u(t) \ast e^{t}u(-t) \quad \text{Where u(t) is the step function}[/tex]

Homework Equations


The Attempt at a Solution



[tex]2e^{t}\int_{?}^{?}e^{-\tau}u(\tau) e^{-\tau}u(-t+\tau)d \tau[/tex]

I'm not sure what the limits on my integral should be, the u(-t) is confusing me. Shouldn't the second function only be nonzero from -infinity to 0 and the other from to +infinity?

Thanks again!
 
Last edited:
Physics news on Phys.org
  • #2
jegues said:

Homework Statement



Convolve,

[tex]2e^{-t}u(t) \ast e^{t}u(-t) \quad \text{Where u(t) is the step function}[/tex]

Homework Equations





The Attempt at a Solution



[tex]2e^{t}\int_{?}^{?}e^{-\tau}u(\tau) e^{-\tau}u(-t+\tau)d \tau[/tex]

I'm not sure what the limits on my integral should be, the u(-t) is confusing me. Shouldn't the second function only be nonzero from -infinity to 0 and the other from to +infinity?

Thanks again!

The limits are from ##-\infty## to ##\infty##, but of course the step functions can cut the interval down by being zero in places. But perhaps an easier approach would be to remember that the Fourier transform ##\mathcal F## satisfies$$
\mathcal F(f * g) =\hat f (\omega)\cdot \hat g(\omega)$$So transform the functions, multiply the transforms, and invert the result. That would give ##f*g## and you can likely just use transform tables and be done with it.
 
  • #3
LCKurtz said:
The limits are from ##-\infty## to ##\infty##, but of course the step functions can cut the interval down by being zero in places. But perhaps an easier approach would be to remember that the Fourier transform ##\mathcal F## satisfies$$
\mathcal F(f * g) =\hat f (\omega)\cdot \hat g(\omega)$$So transform the functions, multiply the transforms, and invert the result. That would give ##f*g## and you can likely just use transform tables and be done with it.

We haven't learned that technique yet nor do we have access to tables so is there a way we can mechanically do the integration?

How would it be done?
 
  • #4
For what values of [itex]\tau[/itex] is [itex]u(\tau)[/itex] nonzero?

For what values of [itex]\tau[/itex] is [itex]u(-t + \tau)[/itex] nonzero?
 
  • #5
jegues said:

Homework Statement



Convolve,

[tex]2e^{-t}u(t) \ast e^{t}u(-t) \quad \text{Where u(t) is the step function}[/tex]

Homework Equations





The Attempt at a Solution



[tex]2e^{t}\int_{?}^{?}e^{-\tau}u(\tau) e^{-\tau}u(-t+\tau)d \tau[/tex]

I'm not sure what the limits on my integral should be, the u(-t) is confusing me. Shouldn't the second function only be nonzero from -infinity to 0 and the other from to +infinity?

Thanks again!

LCKurtz said:
The limits are from ##-\infty## to ##\infty##, but of course the step functions can cut the interval down by being zero in places. But perhaps an easier approach would be to remember that the Fourier transform ##\mathcal F## satisfies$$
\mathcal F(f * g) =\hat f (\omega)\cdot \hat g(\omega)$$So transform the functions, multiply the transforms, and invert the result. That would give ##f*g## and you can likely just use transform tables and be done with it.

jegues said:
We haven't learned that technique yet nor do we have access to tables so is there a way we can mechanically do the integration?

How would it be done?

Well, start with this:$$
2e^{t}\int_{-\infty}^{\infty}e^{-\tau}u(\tau) e^{-\tau}u(-t+\tau)d \tau$$Then, since ##u(\tau)## is zero for ##\tau <0## and 1 for ##\tau > 0## you can write it as$$
2e^{t}\int_{0}^{\infty}e^{-\tau}\cdot 1 \cdot e^{-\tau}u(-t+\tau)d \tau$$Next you know ##u(\tau - t)## is only going to be 1 if ##\tau > t##, so that needs to be taken into account in the limits. When you work on it you will see it matters whether ##t > 0## or not. See if you can take it from there.
 

1. What is a convolution problem?

A convolution problem is a mathematical operation that combines two functions to produce a third function. It is often used in signal processing and image filtering to manipulate data.

2. What do the terms 2e-tu(t) and etu(-t) mean?

The term 2e-tu(t) represents a function that decays exponentially with time, while etu(-t) represents a function that increases exponentially with time. The u(t) and u(-t) terms indicate that the functions are only defined for positive and negative values of time, respectively.

3. How do you solve a convolution problem?

To solve a convolution problem, you need to perform the convolution operation, which involves integrating the product of the two functions over a defined range of values. In this case, it would involve integrating the product of 2e-tu(t) and etu(-t) over the range of time.

4. What is the purpose of solving a convolution problem?

The purpose of solving a convolution problem is to analyze and manipulate data. In signal processing, convolutions can be used to filter out noise or extract specific features from a signal. In image processing, convolutions can be used for tasks such as blurring or edge detection.

5. Are there any practical applications of solving this specific convolution problem?

Yes, this specific convolution problem has practical applications in signal processing and filtering. It can also be used to model certain physical processes, such as the diffusion of particles in a medium.

Similar threads

  • Calculus and Beyond Homework Help
Replies
0
Views
167
  • Calculus and Beyond Homework Help
Replies
1
Views
705
  • Differential Equations
Replies
1
Views
752
  • Engineering and Comp Sci Homework Help
Replies
4
Views
1K
Replies
4
Views
752
  • Calculus and Beyond Homework Help
Replies
4
Views
1K
  • Calculus and Beyond Homework Help
Replies
21
Views
1K
  • Calculus and Beyond Homework Help
Replies
1
Views
1K
  • Calculus and Beyond Homework Help
Replies
14
Views
2K
  • Calculus and Beyond Homework Help
Replies
23
Views
2K
Back
Top